Question and Answers Forum

All Questions      Topic List

Algebra Questions

Previous in All Question      Next in All Question      

Previous in Algebra      Next in Algebra      

Question Number 114720 by Algoritm last updated on 20/Sep/20

Answered by 1549442205PVT last updated on 20/Sep/20

x^4 −2x^3 +4x−2⇔(x^2 −x+1)^2 −3x^2 +6x−3=0  ⇔(x^2 −x+1)^2 −3(x−1)^2 =0  ⇔[x^2 +((√3)−1)x+1−(√3)][x^2 −(1+(√3))x+1+(√3)]=0  i)⇔x^2 +((√3)−1)x+1−(√3)=0  Δ=((√3)−1)^2 +4((√3)−1)=2(√3)  x=((1−(√3)±(√(2(√3))))/2)  ii)x^2 −(1+(√3))x+1+(√3)=0  Δ=(1+(√3))^2 −4(1+(√3))=−2(√3)<0  ⇒has no roots  Thus,the given equation has two  roots x∈{((1−(√3)+(√(2(√3))))/2),((1−(√3)−(√(2(√3))))/2)}

$$\mathrm{x}^{\mathrm{4}} −\mathrm{2x}^{\mathrm{3}} +\mathrm{4x}−\mathrm{2}\Leftrightarrow\left(\mathrm{x}^{\mathrm{2}} −\mathrm{x}+\mathrm{1}\right)^{\mathrm{2}} −\mathrm{3x}^{\mathrm{2}} +\mathrm{6x}−\mathrm{3}=\mathrm{0} \\ $$$$\Leftrightarrow\left(\mathrm{x}^{\mathrm{2}} −\mathrm{x}+\mathrm{1}\right)^{\mathrm{2}} −\mathrm{3}\left(\mathrm{x}−\mathrm{1}\right)^{\mathrm{2}} =\mathrm{0} \\ $$$$\Leftrightarrow\left[\mathrm{x}^{\mathrm{2}} +\left(\sqrt{\mathrm{3}}−\mathrm{1}\right)\mathrm{x}+\mathrm{1}−\sqrt{\mathrm{3}}\right]\left[\mathrm{x}^{\mathrm{2}} −\left(\mathrm{1}+\sqrt{\mathrm{3}}\right)\mathrm{x}+\mathrm{1}+\sqrt{\mathrm{3}}\right]=\mathrm{0} \\ $$$$\left.\mathrm{i}\right)\Leftrightarrow\mathrm{x}^{\mathrm{2}} +\left(\sqrt{\mathrm{3}}−\mathrm{1}\right)\mathrm{x}+\mathrm{1}−\sqrt{\mathrm{3}}=\mathrm{0} \\ $$$$\Delta=\left(\sqrt{\mathrm{3}}−\mathrm{1}\right)^{\mathrm{2}} +\mathrm{4}\left(\sqrt{\mathrm{3}}−\mathrm{1}\right)=\mathrm{2}\sqrt{\mathrm{3}} \\ $$$$\mathrm{x}=\frac{\mathrm{1}−\sqrt{\mathrm{3}}\pm\sqrt{\mathrm{2}\sqrt{\mathrm{3}}}}{\mathrm{2}} \\ $$$$\left.\mathrm{ii}\right)\mathrm{x}^{\mathrm{2}} −\left(\mathrm{1}+\sqrt{\mathrm{3}}\right)\mathrm{x}+\mathrm{1}+\sqrt{\mathrm{3}}=\mathrm{0} \\ $$$$\Delta=\left(\mathrm{1}+\sqrt{\mathrm{3}}\right)^{\mathrm{2}} −\mathrm{4}\left(\mathrm{1}+\sqrt{\mathrm{3}}\right)=−\mathrm{2}\sqrt{\mathrm{3}}<\mathrm{0} \\ $$$$\Rightarrow\mathrm{has}\:\mathrm{no}\:\mathrm{roots} \\ $$$$\mathrm{Thus},\mathrm{the}\:\mathrm{given}\:\mathrm{equation}\:\mathrm{has}\:\mathrm{two} \\ $$$$\mathrm{roots}\:\mathrm{x}\in\left\{\frac{\mathrm{1}−\sqrt{\mathrm{3}}+\sqrt{\mathrm{2}\sqrt{\mathrm{3}}}}{\mathrm{2}},\frac{\mathrm{1}−\sqrt{\mathrm{3}}−\sqrt{\mathrm{2}\sqrt{\mathrm{3}}}}{\mathrm{2}}\right\} \\ $$

Answered by MJS_new last updated on 20/Sep/20

x^4 −2x^3 +4x−2=0  (x^2 −(1−(√3))x+1−(√3))(x^2 −(1+(√3))x+1+(√3))=0  x_(1, 2) =((1−(√3)±(√(2(√3))))/2)  x_(3, 4) =((1+(√3))/2)±((√(2(√3)))/2)i

$${x}^{\mathrm{4}} −\mathrm{2}{x}^{\mathrm{3}} +\mathrm{4}{x}−\mathrm{2}=\mathrm{0} \\ $$$$\left({x}^{\mathrm{2}} −\left(\mathrm{1}−\sqrt{\mathrm{3}}\right){x}+\mathrm{1}−\sqrt{\mathrm{3}}\right)\left({x}^{\mathrm{2}} −\left(\mathrm{1}+\sqrt{\mathrm{3}}\right){x}+\mathrm{1}+\sqrt{\mathrm{3}}\right)=\mathrm{0} \\ $$$${x}_{\mathrm{1},\:\mathrm{2}} =\frac{\mathrm{1}−\sqrt{\mathrm{3}}\pm\sqrt{\mathrm{2}\sqrt{\mathrm{3}}}}{\mathrm{2}} \\ $$$${x}_{\mathrm{3},\:\mathrm{4}} =\frac{\mathrm{1}+\sqrt{\mathrm{3}}}{\mathrm{2}}\pm\frac{\sqrt{\mathrm{2}\sqrt{\mathrm{3}}}}{\mathrm{2}}\mathrm{i} \\ $$

Terms of Service

Privacy Policy

Contact: info@tinkutara.com